Download as pdf
Download as pdf
You are on page 1of 516
Question 1 Question 1 of 193 < > You are working in a medical admissions unit. Following the British HIV Association guidelines which of the following conditions should prompt you to consider performing an HIV test? (Please select 1 option) O Cerebral abscess © Tuberculosis © Allofthe above © Bacterial pneumonia © Lymphadenopathy encaner ss pel All of the above Ueki as a Bacterial pneumonia Lymphadenopathy Key Learning Points Infectious Diseases + Have a low threshold for testing for HIV, be aware of the clinical indicator diseases and that HIV testing does not require extensive pre-test counselling. Explanation In 2012 it was estimated that 21% of people living with HIV in the UK are undiagnosed. The BHIVA 2008 guidelines recommend that in low prevalence areas (where HIV prevalence is less than 2 per 1000) then HIV tests should be performed for a wide range of ‘clinical indicator’ diseases, amongst which are the options listed here. A full list can be found in the guidelines referenced below. Itis in the capability of every doctor, nurse and health care professional to perform an HIV test. HIV is a treatable condition with a normal life expectancy if diagnosed early, and therefore no longer requires pre-test counselling. The patient should be informed of the benefits of testing and how and when the result will be provided to them as with any other medical test. If HIV is detected early it dramatically reduces the morbidity and mortality associated with HIV. Furthermore from a public health aspect it is thought that 50% of new infections occur in people unaware that they have HIV. Question 2 of 193 A 38-year-old Caucasian man with HIV attended the clinic for review. He had been diagnosed with HIV infection three years previously after presenting to hospital with disseminated herpes zoster. His CD4 T cell count at the time of diagnosis was 100 cells/mm? (800-1200). He was started on high-dose valaciclovir and subsequently on highly active antiretroviral therapy consisting of zidovudine, lamivudine and lopinavir. He was also prescribed co-trimoxazole as prophylaxis against Pneumocystis jirovecii pneumonia and fluconazole for oropharyngeal candidiasis. Seven months later, his CD4 count had risen to 210 cells/mm? and his HIV viral load, measured by quantitative PCR, was undetectable. As part of his routine panel of blood tests taken at his three monthly review, fasting serum lipid levels showed: Serum cholesterol 8.1 mmol/L (<5.2) Serum triglyceride 4.2 mmol/L (0.45-1.69) Dietary modifications were made, with which he remained compliant, but despite this a repeat sample taken two months later showed identical values. Which of the following is the best option for treating this patient's dyslipidaemia? (Please select 1 option) O O Fenofibrate Omega-3 fatty acids Pravastatin Ezetimibe Nicotinic acid encane ss el Pravastatin Ezetimibe Nicotinic acid Key Learning Points Cardiology, Infectious Diseases, Lipids, Pharmacology + Lipodystrophy, lipoatrophy and alterations in serum lipid values have been observed in patients with human immunodeficiency virus (HIV) disease taking highly active antiretroviral therapy. Elevated serum lipid levels have been associated with premature coronary artery disease. Explanation Lipodystrophy, lipoatrophy and alterations in serum lipid values have been observed in patients with human immunodeficiency virus (HIV) disease taking highly active antiretroviral therapy. Elevated serum lipid levels have been associated with premature coronary artery disease. The causes of these changes are likely to be multifactorial, but appear much commoner in patients taking protease inhibitors. Patients with predominant. orwith a mixed picture should first be assessed whether dietary modification might be of benefit. hyperc When serum cholesterol (with or without elevated triglyceride) persists, a statin should be used. However, caution must be exercised since some protease inhibitors interact with some statins due to metabolism by CYP3A4 pathway. Simvastatin is contraindicated in patients on protease inhibitors and plasma levels of atorvastatin are also greatly elevated in these patients. For this reason, pravastatin is usually the drug of choice. Ccaeaner ps bl Question 3 of 193 A diplomat with the Commonwealth is going to Uganda to observe the elections. He has had his vaccinations in preparation for the trip. He has previously been treated for schizophrenia but he is now better and is able to work. He is worried about malaria and you advise him about protecting himself from mosquito bites. He wants to know the best medication for prophylaxis against malaria. Which of these drugs would you recommend? (Please select 1 option) oO oO Chloroquine prophylaxis Proguanil prophylaxis Sulfadoxine-pyrimethamine prophylaxis Mefloquine prophylaxis Doxycycline prophylaxis encaner ss pel Proguanil prophylaxis -) Sulfadoxine-pyrimethamine prophylaxis Mefloquine prophylaxis © dogeycineproptvons (Es Explanation Doxycycline prophylaxis is the safest option with less resistance in many parts of the world compared to the other options available. Chloroquine prophylaxis is wrong because of high levels of resistance to the drug around the world. Mefloquine prophylaxis could be used but this patient has a mental illness which is a contradiction for its use. Proguanil prophylaxis is wrong but it can be used in combination with atovaquone for prophylaxis. Proguanil should not be used alone as malaria could develop resistance to it. Atovaquone and proguanil are used for prophylaxis especially where there are high levels of resistance against most of the other drugs. Sulfadoxine-pyrimethamine prophylaxis cannot be used because of the high levels of resistance against the drug. Next Question c encaner ss pel A 27-year-old man was referred to hospital with fevers and haemoptysis. Two weeks earlier he had presented to the Emergency department following a tonic-clonic seizure. Pending the results of sputum cultures, what is the most appropriate combination of antibiotics that should be used to treat this patient initially? ) — Amoxicillin/clavulanate + clarithromycin 2 Flucloxacillin O Azithromycin © Vancomycin + ceftazidime O Cefuroxime + metronidazole Flucloxacillin Azithromycin Vancomycin + ceftazidime Cefuroxime + metronidazole fe Key Learning Points Infectious Diseases, Photographic, Radiology, Respiratory Medicine, Therapeutics * Anaerobic and gram negative organisms are common in abscesses following aspiration. An unconscious patient is at risk of aspiration. Explanation The slide shows an abscess in the right mid-zone. The lung abscess is likely the result of aspiration during the tonic-clonic seizure that occurred several weeks earlier. Due to the angle of the bronchi, the right lung is more commonly affected by aspiration than the left lung. Classically, when the subject is lying down, aspirated oral or gastric contents enter the apical segment of the right lower lobe. Anaerobes and Gram negative organisms are the usual organisms in abscesses following aspiration. Question 5 Question 5 of 193 What is the commonest cause for this appearance? (Please select 1 option) Oo Herpes virus infection Oo Systemic lupus erythematosus (SLE) O Ulcerative colitis Oo Streptococcal infection O Salmonella typhiinfection What is the commonest cause for this appearance? (Please select 1 option) Herpesvirus infection (@IGE3 © | Systemic lupus erythematosus (SLE) Ulcerative colitis @ Streptococcal infection © Salmonella typhiinfection Explanation The slide shows typical target lesions of erythema multiforme (EM). Herpes simplexvirus infection is the commonest cause. Other common causes include infection with Mycoplasma and Streptococci. EM may also: Be idiopathic Be drug-induced (sulfonamides, sulphonylureas, barbiturates) Occur in systemic diseases (SLE, inflammatory bowel disease, malignancy). encaner ss pel This 28-year-old man presented to hospital after becoming progressively more breathless over the preceding day. He had developed a dry cough and reported expectoration of bright red blood. He gave a history of malaise and low-grade fever for five days. The rash (pictured) had appeared three days before presentation. What is the most appropriate treatment to start? (Please select 1 option) O O Intravenous aciclovir Co-amoxiclav + clarithromycin High-dose corticosteroids Rifampicin + isoniazid + pyrazinamide + ethambutol Intravenous benzylpenicillin + flucloxacillin pyrazinamide + ethambutol Intravenous aciclovir Co-amoxiclay + clarithromycin High-dose corticosteroids Rifampicin + isoniazid + pyrazinamide + ethambutol Intravenous benzylpen ethambutol in + flucloxacillin pyrazinamide + Key Learning Points Dermatology, Infectious Diseases, Photographic, Respiratory Medicine, Therapeutics + Varicella pneumonia occurs in up to 20% of adults with chickenpox, appearing three to five days into the course of the illness. In adults with pneumonitis, treatment with aciclovir is warranted. Explanation The slide shows the typical rash of chickenpox. Varicella pneumonia occurs in up to 20% of adults with chickenpox, appearing three to five days into the course of the illness. Symptoms include: * Tachypnoea * Cough * Dyspnoea * Fever. Cyanosis, pleuritic chest pain and haemoptysis are common. In adults with pneumonitis, treatment with aciclovir is warranted. encaner ss pel Question 7 Question 7 of 193 Core Questions v < > A 37-year-old intravenous drug user, not receiving any medication, presents with fever, dyspnoea, night sweats, weight loss and non- productive cough of several weeks in duration. He has dry rales on physical examination, and chest radiography reveals diffuse interstitial infiltrates. On further investigation, he is found to be HIV- positive, and the CD4 cell countis 95 cells/mm. Induced sputum is negative for Pneumocystis carnii. Which of these investigations would you do to confirm the diagnosis? (Please select 1 option) O © Gallium scanning Oo Bronchoalveolar lavage oO Open lung biopsy Oo Pulmonary function test O Transbronchial biopsy encaner ss pel Bronchoalveolar lavage Open lung biopsy Pulmonary function test Transbronchial biopsy Key Learning Points Infectious Diseases * Bronchoalveolar lavage is the gold standard for diagnosis of PCP pneumonia. Explanation The sensitivity after using induced sputum is about 70% but bronchoalveolar lavage increases that sensitivity to about 94%. If the bronchoalveolar lavage is still negative then a transbronchial biopsy should be done mainly to look for other diagnoses. Pulmonary function test especially low diffusing capacity of the lung for carbon monoxide (DCO) can support the diagnosis of Pneumocystis jiroveci pneumonia but cannot confirm the diagnosis because other diseases can also cause alow DCO. Gallium scan increased uptake also only supports the diagnosis but cannot be used for diagnosis. Transbronchial biopsy is invasive despite having a high sensitivity. It should be done after inducing sputum and bronchoalveolar lavage has been done. Open lung biopsy is quite invasive despite having sensitivities and specificities of up to 100%. It should be done after inducing sputum and bronchoalveolar lavage has been done. Ccaeaner ss Question 8 of 193 < > 30-year-old male HIV positive patient is admitted with Pneumocystis Jirovicipneumonia (PCP). He stopped taking co-trimoxazole therapy three weeks previously due to a rash. Blood gases reveal: pH 7.19 (7.36-7.44) pCO 5.3 kPa (4.7-6.0) po2 7.3 kPa (11,3-12.6) Which of the following drug combinations would you select as treatment for this patient? (Please select 1 option) Oo High dose Co-trimoxazole and prednisolone © Infiximab, Ticarcillin and iv pentamidine © Ciproftoxacin, metronidazole and nebulised pentamidine Oo Clindamycin, primaquine and prednisolone Oo Rifampicin and isoniazid encaner ss pel Clindamycin, primaquine and prednisolone futon Rifampicin and isoniazid Key Learning Points Emergency Medicine, Infectious Diseases + Treatment of PCP includes co trimoxazole, or iv pentamidine, or clindamycin & primaquine. Steroids should be started in conjunction. Explanation This patient has severe PCP suggested by the pO2 below 10. As he is allergic to co-trimoxazole alternative therapy would be IV Pentamidine or clindamycin with primaquine. Steroids have been shown to reduce mortality and prevent lung damage in people with moderate-to-severe PCP, (The severity is determined on the basis of arterial blood gas results. These agents are used as adjunctive initial therapy only in patients with HiV infection who have severe PCP as defined by a room air arterial oxygen pressure of less than 9 kPa (70 mmHg) or an arterial-alveolar O2 gradient that exceeds 4.5 kPa (35 mmHg). Itis important that steroids be started right away if they are indicated, because their purpose is to keep people stable during those first few days of treatment. While it is true that long term steroid use is immunosuppressive a 21 day tapering course has been shown to be safe and effective. Next Question encaner ss pel Question 9 of 193 < A 66-year-old retired foundry worker was referred to the medical admissions unit after developing sudden onset of right sided chest pain. For the past 24 hours he has felt unwell, with malaise, headache and myalgias. The GP's letter states that the patient has become mildly confused over the past three to four hours. On examination he is febrile (39°C) and confused. Pulse 62 per minute, blood pressure 110/75. Investigations showed: Haemoglobin 165 g/L (130-180) White cell count 20.1 *109/L @1n Neutrophils 18.5 x10°/L as7 Lymphocytes 0.8 x10°/L (1.5-4) Monocytes 0.8 x10°/L (0-0.8) Platelets 390 x109/L (150-400) Serum sodium 121 mmol/L (137-144) Serum potassium 4.3 mmol/L (.5-4.9) Serum urea 6.2mmol/L (25-75) Serum creatinine 99 mol/L (60-110) Serum bilirubin 7umol/L (1-22) Serum AST 63 U/L (1-31) Serum ALP 100 U/L (45-105) Serum albumin 39 g/L (37-49) ABGs on air: pH 742 (7.36-7.44) po2 9.9 kPa (11,3-12.6) pcO2 3.9 kPa (4.7-6.0) Bicarbonate 22 mmol/L (20-28) Urinalysis: Protein + His chest x ray is shown below. A Which of the following investigations is most likely to establish the identity of the causative organism? (Please select 1 option) O O Blood cultures PCR Chlamydia pneumoniae serology Legionella urinary antigen Immunocytological staining of bronchoalveolar lavage fluid Skip Question Go to Summary encaner ss pel Legionellaurinary antigen Immunocytological staining of bronchoalveolar lavage fluid Key Learning Points Infectious Diseases, Photographic, Radiology, Respiratory Medicine * Diagnosis of legionella pneumionia should be made via clinical history, x-ray and urinary antigen. Explanation This patient's symptoms, signs and investigations are highly suggestive of Legionella pneumonia. The illness may start abruptly with a brief prodrome of malaise, myalgia and headache. High fever and non-productive cough are common and may be accompanied by pleuritic chest pain. Confusion may represent toxic encephalopathy. Amarked neutrophil leukocytosis may be associated with concomitant lymphopenia. Hyponatraemia occurs more commonly than with other pneumonias. Liver function abnormalities are common but non-specific. Proteinuria (sometimes myoglobinuria) is common. Chest x ray usually shows lobar consolidation and progresses to bilateral involvement in 50% of cases. Although the diagnosis may be made by culturing the organism from sputum, tests for Legionella antigens in urine offer a rapid test. Urinary antigen testing is 70-100% sensitive and 100% percent specific. Immunological staining and PCR are not as sensitive or specific as urinary antigen testing. Ccaeaner ss Question 10 of 193 < > A 24-year-old man with chronic renal failure underwent renal transplantation with a matched related donor graft. The immediate post-operative period was uneventful and he was discharged home after two weeks with a fully-functioning graft. One month later he presented to the nephrology clinic with a high fever and he was admitted for further investigations. His full blood count showed a marked leukopenia. During the first evening of his admission, his clinical condition deteriorated rapidly and he became dyspnoeic. On reviewing the other investigations, his liver function tests were noted to be markedly deranged. Which of the following is the most likely cause of his illness? (Please select 1 option) oO Cytomegalovirus © Influenza virus O Coxsackie virus O Adenovirus © Parvovirus B19 [ Skin Oniactian encaner ss pel © Cytomegalovirus Ga Influenza virus © Coxsackie virus 2 Adenovirus O) Parvovirus B19 Explanation Cytomegalovirus (CMY) infection is responsible for a substantial fraction of the morbidity and mortality following organ transplantation. Two main factors determine whether a patient will develop CMV infection after transplantation : 1. Whether the donor or recipient harbours a latent virus capable of reactivation after transplantation 2. The degree of immunosuppression after transplantation. Primary disease is the commonest and most severe type of post- transplant CMV infection. This occurs in individuals who have never been infected with CMV who receive an allograft that contains latent virus from a CMV-seropositive donor. encaner ss pel Question 11 of 193 Which one of the following interventions is most likely to reduce the risk of central venous catheter (CVC) blood stream infection? (Please select 1 option) O oO Adose of intravenous vancomycin prior to insertion Impregnation of the surface of the CVC with chlorhexidine Application of a semi-permeable polyurethane dressing following insertion Application of topical fucidic acid following insertion Adose of intravenous teicoplanin prior to insertion encaner ss pel © Impregnation of the surface of the CVC with chlorhexidine / Application of a semi-permeable polyurethane dressing following insertion Application of topical fucidic acid following insertion ) Adose of intravenous teicoplanin prior to insertion Explanation Administration of prophylactic intravenous or topical antibiotics does not reduce CVC blood stream infection. Although no dressing has been shown to reduce CVC blood stream infection, the application of semi-permeable polyurethane dressings is widely recommended. Catheter impregnation reduces the risk of CVC blood stream infection and catheter colonisation. Further Reading: Smith RN, Nolan JP. Central venous catheters. BM/. 2013;347:f6570. encaner ss pel Question 12 of 193 < A 66-year-old retired foundry worker was referred to the medical admissions unit after developing sudden onset of right sided chest pain. For the past 24 hours he has felt unwell, with malaise, headache and myalgias. The GP's letter states that the patient has become mildly confused over the past three to four hours. On examination he is febrile (39°C) and confused. Pulse 62 per minute, blood pressure 110/75 mmHg. Investigations show: Haemoglobin 165 g/L WBC 20.1 *10°/L Neutrophils 18.5 *10°/L Lymphocytes 0.8 x10°/L Monocytes 0.8 x10°/L Platelets 390 «10°/L Serum sodium 121 mmol/L Serum potassium 4.3 mmol/L Serum urea 6.2 mmol/L Serum creatinine 99 umol/L (130-180) (4-11) (1.5-7) (1.5-4) (0-0.8) (10-400) (137-144) (3.5-4.9) (2.5-7.5) (60-110) Serum creatinine 99 mol/L (60-110) Serum bilirubin 7 mol/L (1-22) Serum aspartate transaminase 63 U/L (1-31) Serum alkaline phosphatase 100 U/L (45-105) Serum albumin 39 e/L (37-49) pH 742 (7.36-7.44) poz 9.9 kPa/75 mmHg (11,3-12.6) pCcO2 3.9 kPa/29 mmHg (4.7-6.0) HCO3 22 mmol/L (20-28) Urinalysis: Protein (+) His chest x ray is shown below. R Which of the following antibiotic regimes would be most effective in treating this specific infection? (Please select 1 option) Oo Erythromycin and rifampicin © Amoxicllin and Erythromycin © Levofloxacin Oo Doxycycline O Amoxicillin Amoxicillin and Erythromycin imoVeileyciejammm [his is the correct answer Doxycycline Amoxicillin Key Learning Points Infectious Diseases, Photographic, Radiology, Respiratory Medicine « First line treatment for Legionella pneumonia is with macrolides or quinolones. encaner ss pel Explanation This patient's symptoms, signs and investigations are highly suggestive of Legionella pneumonia. The illness may start abruptly with a brief prodrome of malaise, myalgia and headache. High fever and non-productive cough are common and may be accompanied by pleuritic chest pain. Confusion may represent toxic encephalopathy. A marked neutrophil leukocytosis may be associated with concomitant lymphopenia. occurs more commonly than with other pneumonias. Liver function abnormalities are common but non-specific. Proteinuria (sometimes myoglobinuria) is common. Chest x ray usually shows lobar consolidation and progresses to bilateral involvement in 50% of cases. The newer macrolides (especially azithromycin) and the newer quinolones (especially levofloxacin) are effective for treating legionellosis. In comparison with erythromycin, these agents are more potent, have better tissue penetration and significantly less gastrointestinal toxicity. With these more potent antibiotics, legionnaires' disease can be treated with monotherapy. There are some reports that patients who receive levofloxacin defervesce more quickly than those who receive azithromycin, but the difference is not statistically significant. Rifampin combined with erythromycin was previously recommended based on reports that these antibiotics in combination were more effective than erythromycin alone. However, combination therapy is now only recommended in patients who are failing standard therapy. Other drugs that can be used include tetracycline, doxycycline, and co- trimoxazole. Question 13 of 193 < > A 56-year-old woman presents with recurrent episodes of dysuria with localised tingling of the genitalia, and shooting pains. Which of the following is true regarding genital herpes? (Please select 1 option) O Transmission can occur in the absence of lesions Oo Topical aciclovir is the treatment of choice OC Most primary cases present with symptoms © © Recurrent episodes tend to be more severe O Itismost commonly secondary to herpes simplex virus-1 infection encaner ss pel Transmission can occur in the absence oflesions (@ige3 ) Topical aciclovir is the treatment of choice ©. Most primary cases present with symptoms © | Recurrent episodes tend to be more severe © | ttismost commonly secondary to herpes simplex virus-1 infection Explanation Genital herpes is most commonly caused by herpes simplex virus-2 (HSV). Sixty per cent of primary cases are asymptomatic. Recurrent attacks tend to be shorter and less severe. Viral shedding can occur in the absence of lesions. Antiviral treatment reduces the severity of episodes but is not curative. Topical aciclovir is of little benefit. Question 14 of 193 Core Questions v < > 19-year-old woman presents with severe left iliac fossa pain of acute onset. On examination she has a temperature of 38.0°C. Pelvic examination reveals uterine and left adnexal tenderness. Gram stain of the cervical exudates demonstrates Gram negative intracellular diplococci. Which of the following statements applies? (Please select 1 option) oO Contact tracing should be initiated following treatment O Treatmentis contraindicated in the absence of Gram negative diplococci on Gram staining. © _ Disease notification to the Health Protection Unit (HPU) is required O Serologic tests to detect the presence of antibody in the patient's serum is useful for diagnosis © Infection outside the genital tract in rare. encaner ss pel A 19-year-old woman presents with severe left iliac fossa pain of acute onset. On examination she has a temperature of 38.0°C. Pelvic examination reveals uterine and left adnexal tenderness. Gram stain of the cervical exudates demonstrates Gram negative intracellular diplococci. Which of the following statements applies? (Please select 1 option) Contact tracing should be initiated following treatment This is the correct answer Treatment is contraindicated in the absence of Gram negative diplococci on Gram staining. Key Learning Points Infectious Diseases + Contact tracing should be offered following all confirmed sexual transmitted disease and can be done discretely. Explanation The patient has pe gonorrhoea. secondary to Neisseria This is the second commonest sexually transmitted infection (STI) in the United Kingdom. It is seen as Gram negative intracellular diplococcus on Gram staining. The absence of organisms on Gram stain does not exclude infection. Culture is the gold standard diagnostic tool. Patients may present with: * Urethritis * Epididymitis * Endocervicitis, or * Pelvic inflammatory disease. Patients may also be be asymptomatic. Contact tracing is important for treatment of other infected persons and prevention of re-infection following treatment. The presence of antibody in the patient's serum could be an indication of prior infection and is not an appropriate diagnostic test. Disseminated gonococcal infection occurs in approximately 1% of cases and can cause itis, arthralgia, endocarditis, meningitis, and fevers. All women with suspected gonococcal infection should undergo a pregnancy test as neonatal intrapartum infection can cause conjunctivitis neonatorum and disseminated infection. Question 15 of 193 Core Questions , < > 34-year-old lady presents to the Emergency Department. She moved to the UK from India 10 years ago, but moved to the area three years ago. She works as a medical coder in the hospital. She presents with a three-day history of severe generalised headache, a stiff and painful neck, photophobia and fever. Her husband reports she is drowsy and has been confused since that morning. She is generally well and takes no medications. Her only previous medical history is of groin lymphadenopathy that was investigated five years ago. A biopsy was taken which showed a caseating granuloma. She had no further follow-up at that time. Investigations CXR - lung fields clear. No obvious abnormalities Blood tests: Haemaglobin 138 g/L White cells <20.3x10%9/L Platelets 310x1049/L. CRP >350 Glucose 6.8 mmol/L Cerebrospinal Fluid 20 cmCSF (CSF): Opening pressure Protein 3280 mg/dL White cells 466 cell/uL (polymorphonucleocyte 280 cell/uL and lymphocytes 206 cell/iL) Red cells 4cell/uL Glucose 1.6 mmol/L. Lactate 5.6 mmol/L CSF appears turbid CSF gram stain and PCR - pending What is the best therapy regimen for this patient? (Please select 1 option) O await gram stain and viral PCR prior to starting a targeted therapy O _ Startaciclovir, ceftiaxone and vancomycin © _ Startisoniazid, rifampicin, pyrazinamide and ethambutol as well as ceftriaxone, vancomycin and dexamethasone © Startisoniazid, rifampicin, pyrazinamide and ethambutol O © Start empirical ceftriaxone, vancomycin and dexamethasone Submit Answer Start isoniazid, rifampicin, pyrazinamide and ethambutol as well as ceftriaxone, vancomycin and dexamethasone foe Start isoniazid, rifampicin, pyrazinamide and ethambutol O Start empirical ceftriaxone, vancomycin and dexamethasone Explanation This lady could have either bacterial meningitis or TB meningitis from her CSF results. Although she has no chest signs of previous TB, she has had a previous biopsy suggesting she may have TB in her lymphatic system. This makes TB meningitis the most likely diagnosis, however, treatment should certainly be started empirically. Since the gram stains and stain for acid fast bacilli would not yet be back from the lab, treatment should be started for both bacterial and TB meningitis. References: BRITISH INFECTION SOCIETY GUIDELINES: British Infection Society, guidelines for the diagnosis and treatment of tuberculosis of the central Nervous system in adults and children Question 16 of 193 < > Bactericidal antibiotics should be used to treat infections of the endocardium and meninges if possible. Host defences are relatively ineffective at these sites and the dangers imposed by such infections require prompt eradication of these organisms. Which of these antibiotics is bactericidal? (Please select 1 option) Oo Azithromycin Oo Trimethoprim O Gentamicin Oo Clindamycin Oo Doxycycline encaner ss pel Trimethoprim Gentamicin O Clindamycin O Doxycycline Explanation Gentamicin is bactericidal and the other options are bacteriostatic. Next Question encaners eal A 22-year-old female student presents with generalised pruritus of six weeks duration. She has little other history of note and has otherwise been well. This itching has deteriorated over this time and is particularly worse at night. She has been unaware of any rashes and denies taking any prescribed drugs. There is no history of atopy. She shares a flat with her boyfriend and drinks approximately 12 units per week and smokes cannabis occasionally. On examination, there is little of note except there are some scratch marks. over the shoulders and back and she has some erythema between the fingers. Otherwise cardiovascular, respiratory and abdominal examination is normal. Which of the following therapies would be most appropriate treatment for this patient? (Please select 1 option) O Astemizole Oo Chlorpromazine O Calamine lotion © Permethrin cream Oo Ciprofloxacin Chlorpromazine C Calamine lotion © Se Correct © Ciprofloxacin Explanation This patient has scabies, a highly contagious disease caused by the mite Sarcoptes scabiei, and is quite common in the sexually active. Itis well recognised to cause generalised pruritus and one should make a careful examination between the finger spaces for burrows. Appropriate treatment includes permethrin cream, topical benzyl benzoate, and malathion. The permethrin cream doesn't have any direct effect on the pruritis itself but helps to settle symptoms indirectly by killing the mite, which is the root cause. You should counsel your patients that it may take longer for the itching to settle as the allergic reaction to the mite abates and that the cream should be appplied everywhere below the neck, not merely where there is rash present. Next Question Go to Summary encaner ss pel Question 18 of 193 < > 25-year-old gentleman with Burkitt's lymphoma is admitted and commenced on induction chemotherapy. Within 48 hours it is noticed that his urine output is dropping to 20 mls/hr. Further investigation shows: Potassium 6.5 mmol/L. (3.5-4.9) Calcium. 1.5 mmol/L. (2.2-2.6) Phosphate 4mmol/L (0.8-1.4) Creatinine 250 mol/L (60-110) Which of the following is the most appropriate management of this complication? (Please select 1 option) © Intravenous hydration with 3 litres per day Oo Urinary alkalinisation © Intravenous hydration with 3 L/m? and rasburicase Oo Allopurinol and intravenous hydration Oo Rasburicase only Urinary alkalinisation Intravenous hydration with 3 L/m? and rasburicase Allopurinol and intravenous hydration Rasburicase only Key Learning Points Haematology, Infectious Diseases + Risk factors for ATLS include high grade tumours and large tumour burden. Treatment is with raspuricase. Explanation This question concerns the management of acute tumour lysis syndrome (ATLS). eis acommon complication of HeeriatDIObical tumours which have a high proliferation index, for example, Burkitt's lymphoma, hyperleukocytic acute myelogenous leukaemia (AML), diffuse large B-cell lymphoma. Management revolves around institution of aggressive hydration, aiming for 3 L/m? control of electrolyte disturbances (typically, hypocalcaemia, hyperphosphataemia, hyperkalaemia and uraemia) and clearance of the increased metabolic load with rasburicase, a specific recombinant enzyme. The incorrect answer options omit important aspects of the holistic management of acute tumour lysis syndrome. Ccaeaner ss Question 19 of 193 Core Questions A 39-year-old Ghanaian man presents to the Emergency department following his first grand-mal seizure. He has recently returned from a three-month holiday in Ghana where he was diagnosed with an ischaemic stroke after he developed right sided weakness. He gives a further history of low grade fevers and a dry cough on and off over several months. On examination his GCS is 15/15. His blood pressure is 110/65 mmHg. Neurological examination revealed a mild right hemiparesis. There was no meningism or papilloedema. He undergoes a magnetic resonance scan of the brain that demonstrates several ring enhancing lesions. What is the most likely diagnosis? (Please select 1 option) O cNStuberculosis O- cw encephalitis Oo Primary CNS lymphoma Oo Progressive multifocal leucoencephalopathy Oo Toxoplasmosis Toxoplasmosis Key Learning Points Infectious Diseases * Toxoplasmosis in association with HIV often involves focal neurological deficits with signs of raised intracranial pressure. MRI shows ring enhancing lesions. Explanation From the history given, itis likely that this patient has toxoplasmosis, a central nervous system disease most commonly seen in HIV. The history of low grade fevers and a generalised malaise for months, as well as a clinical suspicion of toxoplasmosis should prompt an HIV antibody test. The findings of several ring enhancing lesions on MRI combined with low grade pyrexia, focal neurological deficit and seizures is most suggestive of toxoplasmosis. I I is usually occurs in the presence of a CD4 count <200/mm3. Progressive multifocal leucoencephalopathy is caused by the JC virus and usually occurs in the presence of CD4 count <1 00/mm?, MRI shows multifocal lesions in the subcortical white matter. Primary CNS lymphoma should be suspected if the patient has no response to anti-toxoplasma therapy. It usually presents with altered mental status. CNS tuberculosis most commonly occurs when the CD4 countis <100/mm:. It presents with headache, fever, meningism, and impaired conscious level. Brain imaging shows meningeal enhancement and occasionally, tuberculomas. Diagnosis is by CSF culture for Mycobacterium tuberculosis. CMV encephalitis presents with headache, fever, photophobia, nuchal rigidity and delirium. A 78-year-old driver with a history of chronic obstructive pulmonary disease was admitted following a road traffic accident in which he sustained severe head injury. He was transferred to the intensive care unit where he underwent tracheal intubation and ventilation. Five days after admission he developed haemoptysis and on examination was found to have coarse crackles in the right lower lung. ‘A sample from tracheal aspirates was sent to the microbiology laboratory. The results are as shown. White blood cell 9,000 Neutrophils 75% Lymphocytes 24% Organisms Gram-negative bacilli Which of the following is the most likely causative organism? (Please select 1 option) Oo Stenotrophomonas maltophilia O Haemophilus influenzae Oo Streptococcus pneumoniae O Mycoplasma hominis Oo Staphylococcus aureus C Stenotrophomonas maltophilia Haemophilus influenzae (ESD O Streptococcus pneumoniae ¢ Mycoplasma hominis \ Staphylococcus aureus Explanation Haemophilus influenzae is a common causative agent for ventilator- associated pneumonia. It is a Gram negative bacillus. It is found in increased frequency in smokers and patients with COPD. Infection tends to occur within five days of hospital admission. Streptococcus pneumoniae is a Gram positive diplococcus. Staphylococcus aureus is a Gram positive coccus. Mycoplasma hominis causes infections of the genitourinary tract. Stenotrophomonas maltophilia often colonises endotracheal tubes, and though it can cause ventilator-associated pneumonia, is a rarer cause than H. influenzae. encaner ss pel Question 21 of 193 < > A 32-year-old man was referred to clinic with a ten year history of recurrent chest infections. He was diagnosed with common variable immunodeficiency. Laboratory investigations showed: IgG 65e/L (6.0-13.0) IgA 08 g/L (08-3.0) lgM OSe/L (0.4-2.5) Which of the following options would best prevent recurrent chest infections? (Please select 1 option) © Intermittent courses of antibiotics when the patient feels he is developing a chest infection Oo Heart-lung transplantation O “Intravenous immunoglobulin Oo Chest physiotherapy and postural drainage Oo Lifelong prophylactic antibiotics encaner ss pel Heart-lung transplantation Intravenous immunoglobulin Chest physiotherapy and postural drainage Lifelong prophylactic antibiotics Key Learning Points Haematology, Immunology, Infectious Diseases, Respiratory Medicine * Common variable immunodeficiency (CVID) is the most prevalent of the primary immunodeficiency diseases and patients are deficient in both IgG and IgA. Explanation Common variable immunodeficiency (CVID) is the most prevalent of the primary immunodeficiency diseases. The basic defect in CVIDis a failure of B lymphocyte differentiation into plasma cells that produce the various immunoglobulin (Ig) subtypes. Patients with CVID often have marked reduction in serum levels of both IgG and IgA; approximately 50% of these patients also have reduced IgM. However, some patients have normal IgG levels but functional antibody deficiency may be present despite normal IgG subclass levels. The diagnosis is based on exclusion of known causes of defects of the humoral immune system. Most cases are sporadic, although familial cases exist that have various inheritance modes (including autosomal dominant with variable penetrance, autosomal recessive, and X linked). Regular intravenous immunoglobulin (1gG) infusions greatly reduce the frequency of recurrent respiratory tract infections. caeanar ss ba Clinical manifestations of CVID include: 1. Recurrent infections: Principally recurrent pyogenic infections of upper and lower respiratory tract. This is the main clinical manifestation of CVID. Symptoms may appear during childhood or, more often, after adolescence. Bronchiectasis may develop if optimal therapy is delayed. Haemophilus influenzae, Moraxella catarrhalis, Streptococcus pneumoniae, and Staphylococcus aureus are the organisms most commonly involved. Severe, recurrent infection with herpes simplex is also common. 2. Autoimmune diseases: Patients may develop: * Thrombocytopenia * Neutropenia * Thyroid abnormalities Vitiligo, and * Keratoconjunctivitis sicca. About 20% of these patients have a severe gastroenteropathy with severe malabsorption resembling coeliac sprue, nodular lymphoid hyperplasia, and chronic inflammatory bowel disease such as ul colitis and Crohn's disease. Asmaller number of patients develop: and pernicious anaemia * Primary biliary cirrhosis * Alopecia totalis * Hyperthyroidism * Vasculitis, and * Lymphoid interstitial pneumonia. encaner ss pel 3. Lymphoid hyperplasia and granulomatous diseases: Atypical lymphoid hyperplasia due to clonal expansion of B or T lymphocytes has been reported in as many as one third of patients with CVID. Granulomas have been reported in approximately 5-10% of patients with CVID. Granulomas are indistinguishable from those of classic sarcoidosis and are found in the: * Lung * Liver * Spleen, and * Conjunctivae. 4. Predisposition to malignancy: Patients with CVID have a high risk of developing malignant neoplasms (most commonly lymphoma, especially non-Hodgkin's lymphoma [NHL], and gastrointestinal [Gl] carcinoma). Most lymphomas are of these are of the B cell immunophenotype and are frequently associated with Epstein-Barr virus (EBV). Lymphoma occurs 300 times more frequently in women with CVID than in affected men. Other malignancies include: * Colon cancer * Breast cancer * Prostate cancer * Ovarian cancer * Oral cancer, and * Melanoma. WUCsuyil ss Question 22 of 193 < > A 26-year-old homosexual man presented to hospital with a fever, dry cough and dyspnoea. Oropharyngeal candidiasis was noted. He was found to be HIV antibody positive and Pneumocystis jiroveciiwas found on bronchoalveolar lavage. His Pneumocystis jirovecii pneumonia (PCP) was treated with intravenous co-trimoxazole and he was subsequently started on highly active antiretroviral therapy (HAART). Four weeks later he presented to clinic complaining of weakness and generalised aching, He was taking highly active antiretroviral therapy (zidovudine (AZT), lamivudine (3TC), nelfinavin), co-trimoxazole as Pneumocystis jiroveciiprophylaxis and fluconazole. Investigations: Serum creatine kinase 6700 IU/L (24-195) Myoglobin found in urine. What is the cause of this problem? (lease select 1 option) O Zidovudine AZD O Lamivudine (3TC) O | Fluconazole O Cotrimoxazole O Nelfinavir eee Zidovudine AZ) (@EEEED Lamivudine (3TC) Fluconazole Co-trimoxazole Nelfinavir Explanation Side-effects of zidovudine (AZT) include: * bone marrow suppression * macrocytosis (can be used as a parameter to monitor adherence to therapy) * myalgia, and * myopathy. Lactic acidosis and lipodystrophy are thought to be side-effect common to all nucleoside analogues. In this case the CK is elevated as the patients going into a picture of rhabdomyolysis, a known assocation with AZT treatment. encaner ss pel Question 23 of 193 Core Questions v < > ‘An 18-year-old male presents to the general medical take. He presents with large grey lesions around his perineum and in his mouth which started two days ago. He reports that he has a more general rash on his trunk and the palms and soles of his feet. He generally feels unwell with malaise and fever. He has never had anything like this before but mentions that he did see a small ulcer on his scrotum a month ago that didn't hurt and healed on its own. He reports that he is generally fit and well and has just moved to the area to. start his university studies. On examination, he has a symmetrical, widespread, Maculo-papular rash on his trunk, limbs and the palms and soles of his feet. This is different from the large greyish lesions that you see in his mouth and perineum. He has general lymphadenopathy that is minimally tender on palpation. His chest is clear on auscultation, there are no added heart sounds and his abdomen is soft and non-tender. Investigations show: Haemoglobin 13¢/dl WCC 11x 1049/1 Platelets 352x10°9/| CRP 89 Sodium 145 mmol/| Potassium 3.8mmol/I Urea 4mmol/| Creatinine 82 mol/| What is the most likely diagnosis? (Please select 1 option) oO oO Treponema pallidum Epstine Barr Virus Cytomegalovirus Herpes Simplex Virus HIV Ccaeaner ss Treponema pallidum Epstine Barr Virus Cytomegalovirus Herpes Simplex Virus HIV Key Learning Points Infectious Diseases * Signs and Symptoms of common sexually transmitted diseases Explanation This young man presents with a symmetrical rash on his trunk, palms. and soles that is pathognomonic for secondary syphilis. He also has a further rash of grey lesion in the mouth and perineum associated with syphilis. Although he was not concerned about it, he has also mentioned that he had a painless genital ulcer one month ago that may have represented his primary syphilis infection. He has general lymphadenopathy and epitrochlear nodes are particularly suggestive of the diagnosis. He is a young man that has recently started university and is likely to be enjoying this period of parties and new acquaintances. It would be worth exploring his sexual history further as syphilis is more common in men who have sex with men. Patients are often co-infected with other sexually transmitted diseases and this patient should be further investigated for these. ~ The structure shown below was identified on microscopy of a sputum. sample from a patient who presented with haemoptysis. He has an abnormal chest radiograph. What treatment should be started? (Please select 1 option) oO oO Intravenous amphotericin B Intravenous amoxicillin/clavulanic acid + clarithromycin lsoniazid + rifampicin + ethambutol + pyrazinamide Intravenous vancomycin Intravenous cefotaxime encaner ss pel He has an abnormal chest radiograph. What treatment should be started? (Please select 1 option) Intravenous amphotericinB = (@iZE9) Intravenous amoxicillin/clavulanic acid + clarithromycin ©) Isoniazid + rifampicin + ethambutol + pyrazinamide Intravenous vancomycin © Intravenous cefotaxime Explanation The slide shows the typical morphology of Aspergillus fumigatus. The patient will need treatment with amphotericin. | Go to Summary | encaner ss pel A 16-year-old male presented to hospital with a 24 hour history of fever and confusion. He was known to have epilepsy, which was well controlled on drugs. He also had a previous history of drug overdose. He consumed 50 units of alcohol per week and admitted to using recreational drugs. On examination his temperature was 39°C. Investigations showed: Haemoglobin 10g/L (130-180) White cell count 15 x10°/L (4-11) Platelets 156 x109/L (150-400) Serum sodium 134 mmol/L (137-144) Serum potassium 4.1 mmol/L (3.5-4.9) Serum urea 12 mmol/L (2.5-7.5) Serum creatinine 160 mol/L. (60-110) Serum ALT TIOU/L (5-35) Serum AST 200 U/L. (1-31) Serum ALP 120 U/L (45-105) Serum bilirubin 20 umol/L (1-22) Serum GGT 400 U/L. (<0) Urine Protein + His chest x ray on admission is shown. What is the best combination of antibiotics in the first instance? (Please select 1 option) O Amoxicillin and metronidazole oO Doxycycline © Ampicillin and gentamicin oO Erythromycin and rifampicin O Cefotaxime and erythromycin fweeer] FN What is the best combination of antibiotics in the first instance? (Please select 1 option) Amoxicillin and metronidazole Doxycycline Ampicillin and gentamicin Erythromycin and rifampicin Cefotaxime and erythromycin Key Learning Points Infectious Diseases, Photographic, Radiology, Respiratory Medicine * Epilepsy and alcohol and drug abuse are risk factors for aspiration pneumonia. Explanation The diagnosis is aspiration pneumonia. The most frequent conditions associated with aspiration pneumonia are decreased consciousness and dysphagia. He has several risk factors for impaired consciousness * Epilepsy * High alcohol intake and * Use of recreational drugs with a history of drug overdose. Dental sepsis also increases the risk by increasing the anaerobic flora in the mouth and pharynx. Infection, particularly in the community, usually results from anaerobic organisms such as Peptostreptococcus and Bacteroides. Anaerobic infection can result in pneumonia, lung abscesses and empyema. Aspiration pneumonia can be indistinguishable from bacterial community acquired pneumonia in the early stages, as foul smelling sputum does is not usually present until necrosis has occurred. The dependent segments of the lung are usually involved, that is, the posterior segments of the upper lobes and apical segments of the lower lobes when the patient is supine and the lower lobes when the patient is upright. Amoxicillin is appropriate but does not cover all Bacteroides. Metronidazole has good cover against anaerobes but not against aerobes and therefore should not be used alone. encaner ss pel Question 26 Question 26 of 193 < A 45-year-old woman was seen in the Emergency department having developed weakness and stiffness of her legs. Her symptoms had been progressive over a number of months. She appeared to walk with a stiff gait and had had a number of falls. She hada history of joint pains affecting her hands and feet and had recently noticed pain in her back around the thoracic area. She had not noticed any change in her bowels, but had noticed increased urinary frequency. She had attended her GP eight months ago with painful visual disturbance in her right eye, which seemed to recover after three days. No diagnosis was made at the time. Otherwise, she was fit and well, was a non-smoker and did not drink alcohol. On examination she had multiple oral ulcers. Her blood pressure was 145/78 mmHg, pulse was 87 beats per minute and regular and she was apyrexial. Examination on the cranial nerves did not reveal any abnormalities and fundoscopy was entirely normal. Upper limb examination revealed deformities of both hands as shown. Tone, power, reflexes and sensation in the upper limbs were intact. Lower limb examination revealed marked increase in tone with pyramidal weakness of 3/5 in both legs and pathologically brisk reflexes with bilateral extensor plantar responses. There was also reduced sensation to pinprick bilaterally to the level of T4. A photograph of the patient's hands is shown below: Investigations revealed: * Rheumatoid factor negative * Anti-nuclear antigen 1: 160 * Extractable nuclear antigen negative Alumbar puncture was performed and yielded the following data: Opening pressure CSF protein CSF white cell count CSF white cell differential CSF red cell count CSF oligoclonal bands Serum oligoclonal bands 12mH20 079 g/L 10 cells permL 90% lymphocytes 2 cells per mL Positive Positive (6-18) (0.15 -0.45) (<5) (<5) Ccaeanars os Aplain x ray of the hands did not reveal any bone deformity. Whatis the likely diagnosis in this patient? (Please select 1 option) O O Multiple sclerosis Reactive arthritis Myelopathy secondary to rheumatoid arthritis Systemic lupus erythematosus Myelopathy secondary to osteoarthritis encanta ® ‘Systemic lupus erythematosus Myelopathy secondary to osteoarthritis Explanation This patient presents with a subacute myelopathy in the presence of a positive anti-nuclear antigen, lymphocytic pleocytosis and oligoclonal bands. In addition she also has oral ulcers and a Jaccoud's arthropathy. The picture shows joint subluxations and swan neck deformities, caused by recurrent episodes of synovitis that damage tendon sheaths and slings resulting in joint deformity but, in this case, there is no bone deformity. Jaccoud's arthropathy is seen in: © SLE * Rheumatic fever * Parkinson's disease, and * Hypocomplementaemic urticarial vasculitis. Reactive arthritis is a triad of urethritis, conjunctivitis and sero-negative arthritis with no associated neurological sequelae. Next Question | Go to Summary | encaner ss pel A 48- year-old man with diabetes mellitus presents with a 24 hour history of an increasingly painful left leg. le Whatis the most likely causative organism? (Please select 1 option) O _ Group G Streptococcus ©. Staphylococcus epidermidis oO Staphylococcus aureus © Group B Streptococcus © _ Group A Streptococcus Staphylococcus epidermidis Staphylococcus aureus Group B Streptococcus U Group A Streptococcus Explanation Ascending cellulitis is shown. Staphylococcus aureus and Streptococciare the commonest causative organisms. Group B Streptococcus has a predilection for diabetic patients and is the likeliest causative organism in this scenario. Further Reading: Medscape. Streptococcus Group B Infections. | Go to Summary | encaner ss pel < A 27-year-old refugee from Cambodia presents with a short history of fatigue and jaundice. He has lived in the United Kingdom for the last 12 months. His mother died of liver disease age 55 and his older brother died of liver cancer three years previously in Cambodia age 40. His brother was known, to have hepatitis B. He consumes 30-40 units of alcohol a week. Investigations show: Haemoglobin 145 g/L WBC 7.6 x10°/L Platelets 156 *10°/L Serum bilirubin 213 mol/L Serum gamma glutamyl transferase 200 U/L Serum alkaline phosphatase 150 U/L Serum aspartate aminotransferase 912 U/L Serum alanine aminotransferase 1450 U/L Serum alpha-fetoprotein 45 kKU/L HBsAg Positive HBeAg Positive Anti-HBc (lgM) Positive (130-180) (4-11) (150-400) (1-22) (<50) (45-105) (1-31) (5-35) (<10) Anti-HAV (igM) Negative Anti-HAV (IgG) Positive Anti-HCV Negative An ultrasound scan of his liver shows mild hepatomegaly and an echogenic liver but no other abnormality. What is the most likely diagnosis? (Please select 1 option) Oo Hepatocellular cancer (HCC) oO Flare of chronic hepatitis B O Acute hepatitis B O — Alcoholrelated hepatitis Oo Hepatitis E encaner ss pel Flare of chronic hepatitis B Aci e hepatitis B Alcohol related hepatitis Hepatitis E Key Learning Points Gastroenterology, Infectious Diseases « In low risk areas a combination of HBsAg positivity and features of acute hepatitis usually indicates acute self-limiting hepatitis B infection. Explanation In areas of low HBV prevalence, such as the United Kingdom, a combination of HBsAg Coe and features of acute hepatitis usually indicates acute self-limiting In countries with high prevalence of | s B the majority of infection is acquired vertically during childhood and leads to chronicity rather than acute infection. Where there is an intermediate or high level of hepatitis B prevalence flares of chronic hepatitis B and acute self-limiting mutually misdiagnosed where an individual's hepatitis B status is unknown prior to the current illness. Itis frequently very difficult to make the distinction especially on the basis of a single assessment without knowledge of the course of the episode. Chronic hepatitis B is defined by the presence of HBsAg in the serum for more than six months. HBeAg in the serum is present in acute infection and may persist in chronic for several years. caeanar ss ba In chronic infection eAg seroconversion marks a change from the immune tolerance to immune control phase of the disease. HBeAg presence cannot be used to distinguish between acute HBV or a flare of chronic disease. Anti-HBc-lgM is typically found in acute HBV infection, however it can be found in10-15% of patients with chronic HBV. This is especially true when considering acute flares of chronic hepatitis. The sensitivity and specificity for HBc-IgM to distinguish between acute HBV and a chronic flare has been reported as low as 77% and 70% respectively. Using high titres to determine cut-offs (1:10,000 or greater) does improve this significantly however. Flares of chronic HBV are typically associated with higher levels of HBV DNA and AFP than acute self-limiting disease. Additionally flares of chronic HBV tend to be associated with less necroinflammation, and thus ALT tends to be as raised as in acute HBV, but hepatic synthetic dysfunction is more common. In this case the patient originates from an area of the world with a high prevalence of HBV infection. Additionally there is clearly a strong family history of liver disease which, given the clinical context, is highly likely to be attributable to HBV infection. No risk factors for the acquisition of HBV acutely are described. The combination of these factors with the significantly raised AFP and relatively modest elevation of the ALT with marked jaundice would tend to favour a diagnosis of a flare of chronic HBV over acute HBV despite low titre anti-HBc-IgM titre. The alanine aminotransferase (ALT) is too high for alcohol-related hepatitis. The alpha-fetoprotein is commonly elevated during acute hepatitis due to hepatic regeneration. HCC is unlikely in a 27-year-old and would not account for the raised ALT. Hepatitis Eis not endemic in the United Kingdom and is a very unlikely diagnosis. Question 29 of 193 < > A 78-year-old male comes to your practice with a cough of one week's duration, difficulty in breathing, chest pain, chills and sweats. On examination he is in a fairly good general condition. He has a temperature of 38°C and BP 110/70 mmHg. His respiratory rate is 28 breaths/minute. On auscultation he has some crepitations. He has a urea of 4.5 mmol/I (2.5 -7.5 mmol/L). He seems to have a band which shows he is allergic to penicillin. How would you treat this patient? (Please select 1 option) O oral doxycycline 200 mg loading then 100 mg od for one week oO lV ceftriaxone 2 g O Weeftriaxone 2 god plus IV clarithromycin Oo Oral levofloxacin 500 mg od for five days OC Oral doxycycline 100 mg od plus oral clarithromycin 500 mg bd for five days encaner ss pel A 78-year-old male comes to your practice with a cough of one week's duration, difficulty in breathing, chest pain, chills and sweats. On examination he is in a fairly good general condition. He has a temperature of 38°C and BP 110/70 mmHg. His respiratory rate is 28 breaths/minute. On auscultation he has some crepitations. He has a urea of 4.5 mmol/l (2.5 -7.5mmol/L). He seems to have a band which shows he is allergic to penicillin. How would you treat this patient? (Please select 1 option) Oral doxycycline 200 mg loading then 100 mg od for one week aa he correct answer IV ceftriaxone 2 g IV ceftriaxone 2 g od plus IV clarithromycin Key Learning Points Infectious Diseases « Low CURB-65 scores in penicillin allergic patients should be treated with oral doxycycline. Explanation The patient has a CURB-65 score of 1, so he has pneumonia of very low severity. Since he is allergic to penicillin, doxycycline alone will be effective in the treatment of pneumonia in this particular case. IV ceftriaxone 2 g od plus IV clarithromycin 500 mg bd would be effective but unnecessary. This combination is used to treat high severity pneumonia. IV ceftriaxone 2 g od is the wrong answer. The patient has low severity pneumonia so there is no need to use an intravenous route for treatment. Oral levofloxacin 500 mg od for five days could be effective but it is unnecessary. It is used to treat moderate severity pneumonia. Oral doxycycline 100 mg od plus oral clarithromycin 500 mg bd for five days could be effective but is unnecessary. It is used to treat moderate severity pneumonia. A 17-year-old male comes to see you with a three day history of sore throat and lassitude. During your discussion he says that preceding these acute symptoms he has been feeling unwell for a week or so. On examination he appeared well but anxious. His tonsils were mildly inflamed and there were a few palpable cervical lymph nodes. Abdominal examination was unremarkable. You tell him that you think that he has a viral illness but arrange a number of fasting blood tests mainly for reassurance. When he returns for review you note from the blood tests that he is jaundiced. Investigations show: Haemoglobin 1528/L MCV 92 fL White cell count 7.2*10°/L Platelets 310 «10°/L Reticulocytes 1% Plasma bilirubin 60 umol/L Plasma conjugated bilirubin 4umol/L Plasma aspartate transaminase (AST) 19 U/L Plasma alkaline phosphatase 105 U/L Plasma albumin 42e/L (130-180) (80-96) (4-11) (150-400) (0.2-2.0) (1-22) (0-4) (S-35) (45-105) (37-49) What is the most likely explanation for the abnormal liver function tests? (Please select 1 option) e) O Chronic hepatitis C infection Epstein-Barr virus infection Primary biliary sclerosis Acute hepatitis A infection Gilbert's syndrome Skip Question Go to Summary encaner ss pel Gilbert's syndrome Explanation The laboratory tests show elevated unconjugated bilirubin with otherwise normal liver function tests. The hyperbilirubinaemia has occurred after an illness (likely viral) associated with reduced calorie intake. Plasma levels of unconjugated bilirubin double when calorie intake falls. This may increase bilirubin levels above the clinically detectable threshold (40 mmol/L) in patients with Gilbert's syndrome. encanta < A 15-year-old girl presented to the Emergency department with difficulty breathing. She was a known asthmatic who had seen her general practitioner the previous day with a sore throat. He had diagnosed tonsillitis and had prescribed a 5 day course of oral amoxicillin. The patient had been diagnosed with ulcerative colitis three years previously. Her regular medication included inhaled salbutamol and beclomethasone and mesalazine 400 mg tds. On examination she was alert and oriented but was in considerable distress with laboured breathing. Inspiratory wheeze was noted. She was pale, sweaty and cyanosed. Her temperature was 36.5°C, pulse 1 20/minute and regular, blood pressure 90/35 mmHg. The lungs were clear and the remainder of the examination was normal. She was given high-flow oxygen through a face mask but despite this her breathing became increasingly difficult. What is the most likely pathogen? (Please select 1 option) Oo Streptococcus pneumoniae O Haemophilus influenzae O Staphylococcus aureus © Corynebacterium diphtheriae © Peptostreptococcus encaner ss pel Haemophilus influenzae @SOE ee So Staphylococcus aureus Corynebacterium diphtheriae Peptostreptococcus Explanation The most likely diagnosis is acute epiglottitis. Sudden airway obstruction may occur and it is vital to obtain the assistance of an anaesthetist urgently. No attempt should be made to visualise the epiglottis until an anaesthetist is present as there is a high risk of causing acute airway obstruction by touching the inflamed tissue. The diagnosis may be confirmed on direct visualisation of a cherry-red epiglottis. Early intubation is essential, especially in cases where there is respiratory distress. Adult epiglottitis is much less common but has a higher mortality. The usual causative organism is Haemophilus influenzae type b. Question 32 of 193 < A 23-year-old student is referred by his G.P for investigation of a six month history of malaise, anorexia, and weight loss. He also describes a four week history of diarrhoea. On examination he is cachectic. On inspection of his oral mucosae he has white frond-like patches on both lateral margins of his tongue. Attempts to scrape off the patches are unsuccessful. What is the causative organism of the abnormality on his tongue? (Please select 1 option) oO O° Epstein-Barr virus Candida albicans Herpes simplex virus Human herpes virus 8 Tinea rubrum encaner ss pel © Epstein-Barr virus Gz & Candida albicans O Herpes simplex virus 4 Human herpes virus 8 Tinea rubrum Explanation This describes oral hairy leukoplakia, a condition seen in HIV-infected patients with a CD4 count between 200 and 500/mm3. The condition is painless, however, unlike Candida, the lesions cannot be scraped off the tongue. The diagnosis is a clinical one. As itis asymptomatic, treatment is rarely required. The condition responds to highly active antiretroviral therapy (HAART). Human herpes virus 8 (HHV8) is the causative organism for Kaposi's sarcoma. Tinea rubrum causes fungal skin and nail infections. Next Question encaner ss pel Question 33 of 193 Core Questions: y < > A 22-year-old student presents with a two week history of vulval itching, dyspareunia, and a greenish malodorous vaginal discharge. She has a history of type 1 diabetes mellitus. She has had unprotected sexual intercourse several times over the last few weeks. On examination her cervix is erythematous with petechial lesions. The pH of the discharge is 7.0. Genital wet prep shows a high number of white blood cells and a protozoan organism. Which of the following is the most appropriate management? (Please select 1 option) O Amoxicillin O — Miconazole O Metronidazole OT methoprim O © Fluconazole { a encaner ss pel © | Miconazole Metronidazole (53 Trimethoprim © Fluconazole Explanation Trichomonas vaginalis causes: * Itching * Burning * Dyspareunia, and * Dysuria. Itis an anaerobic protozoan which thrives in more alkaline conditions. The pH of the discharge is greater than 6.5. T. vaginalis does not have a cyst form and therefore is transmitted directly, for example, through sexual transmission. It causes inflammation of the vaginal endometrium and therefore makes it more susceptible to other sexually transmitted infections. Treatment of T. vaginalisis with metronidazole. Sexual partners should also be treated, as they may be asymptomatic carriers. encaner ss pel Question 34 of 193 < > A 38-year-old male who is the brother of the British High Commissioner in Kampala, Uganda went to visit his brother about eight months ago fora two week holiday. He took mefloquine for prophylaxis and he defaulted on the medications. Now he reports having fevers, chills, rigours and headaches. You suspect malaria and send his blood slides to the laboratory. The thick blood slide shows parasites that are large with fragmented cytoplasm. The thin film shows enlarged red cells containing amoeboid parasites with Schuffner's nodes. Which of these malarial parasites is likely to be causing the disease? (Please select 1 option) OP. knowlesi O Povale O ep falciparum O Pmatariae O- P.vivax encaner ss pel P. malariae P.vivax §it Key Learning Points Infectious Diseases * Schuffner's nodes are characteristic on a thin blood film of P. Viva malaria Explanation The incubation period of P. vivax can go up to six months or more with malaria being caused by hypnozoites. The malaria episode happened eight months after returning from Africa. The picture of the thin film and thick film is diagnostic for P. vivax. P. falciparumis wrong because its incubation is normally around six days though it can go till 14 days or more. Its thin film usually shows many ring forms of crescent-shaped gametocytes. P. knowlesiis wrong because it usually affects apes and monkeys and is usually isolated in the forests around South East Asia. P. ovaleis wrong because it is quite rare. The incubation period is similar to that of P. vivax but on the thick film the parasites are more compact and smaller. On the thin film the red blood cells appear oval with ragged ends. P. malariae is also wrong because itis rare. Its incubation could go up to14 days like P. falciparum. The thick film will show a few compact tings or small neat schizonts or small round gametocytes with yellow- brown pigment. The thin film will show red blood cells in band forms. Question 35 of 193 A 22-year-old lady, with lupus and antiphospholipid syndrome presented to the Rheumatology clinic with sharp chest pain across her right side. This pain had started three days prior to presentation. She had some shortness of breath and reported that she has a vesicular rash that had started one day ago over that area. She weighs 65 kg and her medications are: Methotrexat 20 mg once weekly Prednisolone 30 mg once a day Aspirin 75 mg once a day Folic Acid 5S mg once a day She was admitted to rheumatology and underwent a CTPA which showed no pulmonary emboli, but widespread bilateral changes of subsolid nodules and ground-glass opacification. Her shortness of breath started to get worse and her observations were taken: temperature 39.1 °C, blood pressure 107/55, heart rate 122, oxygen saturation 88% on air. What is the best treatment for her likely diagnosis? (Please select 1 option) O oO Treat with 1 g intravenous amoxicillin 8 hourly Treat with 10,000 units dalterparin subcutaneously Treat with aciclovir 650 mg intravenous 8 hourly Treat with 1 g intravenous amoxicillin 8 hourly and 500 mg oral clarythromycin 12 hourly Treat with 800 mg aciclovir orally 5 times daily encaner ss pel ‘0 Treat with 1 g intravenous amoxicillin 8 hourly Treat with 10,000 units dalterparin subcutaneously Oo Treat with aciclovir 650 mg intravenous 8 hourly Treat with 1 g intravenous amoxicillin 8 hourly and 500 mg oral clarythromycin 12 hourly Treat with 800 mg aciclovir orally 5 times daily Explanation The most likely diagnosis for this immunocompromised lady with sharp. pain and a vesicular rash is varicella-zoster chickenpox. She is systemically unwell with it and has widespread pulmonary changes that are consistent with varicella pneumonitis. The treatment of choice in an immunocompromised host with varicella-zoster is intravenous aciclovir at a dose of 10 mg/Kg. References: UpToDate: Treatment of varicella (chickenpow infection Next Question encaner ss pel Question 36 of 193 An 85-year-old lady is admitted from a residential home with an eight hour history of confusion and diarrhoea. No additional history is available from the patient. A carer from the home who has accompanied the patient to hospital says that the patientis usually alert and self-caring, though the general practitioner has been called to see her twice recently because she has complained of dysuria and foul-smelling diarrhoea. On examination, the patient is afebrile. She has a distended tender abdomen with no bowel sounds. You ask a surgical colleague to review her. She is subsequently taken to the operating theatre for an exploratory laparotomy. Asection of bowel is excised (shown below): What is the diagnosis? (Please select 1 option) O Multiple colonic adenomata O Pneumatosis coli © Pseudomembranous colitis © Ischaemic colitis O Angiodysplasia encaner ss pel MWledse seiect | Opuony Multiple colonic adenomata Pneumatosis coli Pseudomembranous colitis (@@igs9 > Ischaemic colitis ) Angiodysplasia Explanation The picture shows a thick pseudomembrane adherent to the colonic mucosa. Diarrhoea due to Clostridium difficile has a characteristic foul smell and is usually managed conservatively with oral metronidazole/vancomycin. If associated with pseudomembranous colitis the mortality risk is greatly increased. Next Question | Go to Summary encaner ss pel 47-year-old Turkish man presented to the Emergency department with this lesion on his arm. He also complained of discomfort in the left axilla. The lesion had developed over a period of two weeks and was painless. There was no past history of note. He had recently returned from a vacation to his parents! goat farm in Turkey. Whatis the diagnosis? (Please select 1 option) oO oO Orf Plague Brucellosis Cutaneous anthrax Pyoderma gangrenosum encaner ss pel Plague Brucellosis Cutaneous anthrax Pyoderma gangrenosum Key Learning Points Dermatology, Infectious Diseases, Photographic * In respect to Anthrax infection cutaneous disease is the commonest form of the infection in humans. Explanation The slide shows the typical appearance of cutaneous caused by Bacillus anthracis. Cutaneous disease is the commonest form of the infection in humans and is usually due to contact with infected animals or animal products. is endemic to herd animals in some parts of the world. Following exposure, the skin lesion evolves over a period of ~2 weeks into a papule, pustule, vesicle and eventually forms an ulcer with a central black eschar. The surrounding skin is usually boggy and oedematous. Lesions are usually painless with tender regional lymph nodes. Lesions heal spontaneously in 80-90% of cases; 10-20% of patients progress and become bacteraemic - associated with a high mortality. Penicillin is effective in treating the infection. Further Reading: BM) Best Practice. Anthrax, Ccaeaner ss < > This 45-year-old white female presented to her general practitioner asking for advice regarding removal of these lesions for cosmetic reasons. What additional investigation should be performed? (Please select 1 option) O | Serum cryptococcal antigen assay Oo Biopsy of lesion for histology oO HIV antibody test Oo Herpes virus serology OQ Tzanck smear Serum cryptococcal antigen assay © Biopsy of lesion for histology HlVantibodytest (EGE Herpes virus serology Tzanck smear Explanation This is a case of molluscum contagiosum. Such umbilicated, pearly papules 2-5 mm in diameter are seen in patients with advanced HIV/AIDS (CD4 count less than 200 cells/mm*). They are caused by a DNA pox virus called molluiscum contagiosum virus (MCV). They commonly occur on the face, especially near the eyelids; they also occur on genitals and trunk. They should be treated with cryotherapy, liquid nitrogen or curettage. Next Questi encaner ss pel ™» This patient has an abnormal chest x ray. What is the most likely diagnosis? (Please select 1 option) oO oO Tuberculosis (TB) Amyloidosis Sarcoidosis Squamous cell carcinoma Secondary syphilis encaner ss pel (Please select 1 option) Tuberculosis (TB) [ Correct } Cc Amyloidosis © Sarcoidosis O Squamous cell carcinoma \) Secondary syphilis Explanation The slide shows cutaneous tuberculosis (lupus vulgaris). Cutaneous TB usually occurs due to spread from an endogenous source; more than 80% of cases occur on the face and neck. Lesions begin as papules and coalesce to form a plaque. The centre of the lesion consists of scar tissue while the lesion extends from the periphery. Apple-jelly nodules are classically described at the margins of the lesions. Next Question encaner ss pel Question 40 of 193 Core Questions < > A72-year-old man was seen in the clinic with profuse diarrhoea for one week, opening his bowels up to seven times a day. For the last two days he had noticed blood in his stools and had developed generalised abdominal pain but no vomiting. His past medical history includes gastritis - for which he takes omeprazole - asthma, and type 2 diabetes mellitus. He had no recent history of foreign travel but had just returned from a two. week holiday to the seaside. Prior to the holiday, he had received a short course of clindamycin from his GP for cellulitis. On examination his abdomen was diffusely tender. He was admitted from clinic for further tests. Haemoglobin 109 g/L (135-180) Platelets 250 «10°/L (150-400) White blood cell count 17.0 x10°/L (3.7-11) Neutrophils 14.5 x10°/L (1.5-6.5) Creactive protein 178 mg/L (0-10) Urea 24 mmol/L (2.1-7.1) Creatinine 130 mol/L (62-106) Which of the following statements is true regarding this patient's diagnosis and management? (Please select 1 option) © __ ELISA-based tests are the most sensitive diagnostic tool in the first instance © Toxin detection from stool is the most widely used diagnostic tool Oo Endoscopy has no role in the diagnostic process O- The patient should be admitted and treated with lV cefuroxime and metronidazole O The causative organism should be identified before commencing treatment f ] encaner ss pel Toxin detection from stool is the most widely used diagnostic tool Endoscopy has no role in the diagnostic process The patient should be admitted and treated with IV cefuroxime and metronidazole The causative organism should be identified before commencing treatment Key Learning Points Infectious Diseases * Patients on antibiotics who develop diarrhoea should be investigated for C. diff. Explanation This patient has Clostridium difficile associated diarrhoea. Clindamycin is one of the most frequently implicated antibiotics and symptoms can occur up to 10 weeks following antibiotic therapy. Toxin detection from stool is the most widely used diagnostic tool. ELISA tests are specific but not as sensitive. Culture is sensitive but often does not differentiate between toxigenic and non-toxigenic strains. Treatment with oral metronidazole should be commenced promptly and confirmation of the organism prior to starting treatment is not necessary. Endoscopy is sometimes used when a rapid diagnosis is required or if the patient has an ileus and is unable to produce stool. encaner ss pel < > A 29-year-old IV drug user presents with a two week history of non- productive cough, fatigue, shortness of breath and fever. On examination, he has a respiratory rate of 28, a heart rate of 90 and was hypoxic on exercise. His blood test results were as follows: Haemoglobin M0 e/L (135-180) White cell count 3 *109/L (4-11) Neutrophils 25 *10°/L (1.5-7.4) Lymphocytes 0.4 x109/L (1.1-4.0) Chest x ray Normal Which of the following investigations would be diagnostic? (Please select 1 option) © Bronchoalveolar lavage O Sputum AFB CO Three sets of blood cultures OHV test O CT pulmonary angiogram encaner ss pel ea iM te Neel? Neutrophils 2.5 *10°/L (1.5-7.4) Lymphocytes 0.4 x10°/L (1.1-4.0) Chest x ray Normal Which of the following investigations would be diagnostic? (Please select 1 option) © Bronchoalveolar lavage Gz ) O) Sputum AFB © Three sets of blood cultures oO HIV test Oo CT pulmonary angiogram encaner ss pel Explanation This patient has Pneumocystis jiroveci, formerly known as Pneumocystis carinii. P. jiroveci, formerly thought to be a protozoan, is an atypical fungus that is not susceptible to antifungals. Transmission is airborne, usually in childhood, with reactivation in immunosuppressive states. Clinical features of infection include * Aninsidious onset of fever * Shortness of breath * Cough (occasionally productive) * Haemoptysis * Exercise induced hypoxia. Although up to 90% of chest x rays in patients with P. jiroveciare abnormal, appearances are often non-specific. Between 10-15% of patients with P. jiroveci have normal chest radiographs and close to 30% have non-specific or inconclusive findings. Diagnosis is by identification of the fungus either by silver staining (with methenamine silver) or by PCR amplification. P. jiroveciis occasionally found in induced sputum, however bronchoalveolar lavage increases the rate of diagnosis. encaner ss pel A 42-year-old Zimbabwean man was admitted to the emergency department after becoming unconscious at Heathrow airport shortly after his arrival in the United Kingdom. His wife said that he had been unwell for several weeks, but had complained of a high fever associated with neck stiffness for the past 24 hours. Microscopy of his CSF is pictured. What organism is shown? (Please select 1 option) O Oo Listeria monocytogenes Candida albicans Cryptococcus neoformans Toxoplasma gondii Pneumocystis jirovecii encaner ss pel Cryptococcus neoformans Toxoplasma gondii Pneumocystis jirovecii Key Learning Points Emergency Medicine, Infectious Diseases, Neurology, Photographic * Diagnosis is by demonstration of C. neoformans in the cerebrospinal fluid, shown on an India ink stain: the thick polysaccharide capsule is highlighted around the cell Explanation Diagnosis is by demonstration of C. neoformans in the cerebrospinal fluid, shown on an India ink stain: the thick polysaccharide capsule is highlighted around the cell (shown in slide). Cryptococcal meningitis is an AIDS-defining illness occurring when CD4 less than 50 cells/mm? and may be associated with a pneumonitis. Cryptococcus can also cause papular skin lesions that resemble molluscum contagiosum. The disease is commoner in African populations. encaner ss pel A 36-year-old man from the United Kingdom is studying for a PhD in nutrition. He has previously been based in China where he was studying varieties of rice. Two months ago while in a rice paddy he felt an itch which later developed a maculopapular rash persisting for five days before resolving. He has currently presented confused and his girlfriend reports he has had a severe headache, worse in the morning with nausea and vomiting. His signs progress and he develops progressive inability to use his left hand, slurred speech, impaired memory and judgment. On examination he looks confused and disoriented. His Glasgow coma scale is 13/15. Verbal 4- Confused and disoriented Motor 5 - Localises painful stimuli Eyes 4- Opens eye spontaneously During the examination he has sudden head and eye movements, smacking of the lips and jerky muscle movements. The rest of the examination is unremarkable. Laboratory results show no ova or cysts in the stool and urine. Full blood count shows eosinophilia which was 600/uL. Other indices are within normal limits. Both haemagglutination and ELISA showed antibodies of Schistosoma and when repeated they were still positive. The CT showed oedema and multifocal small contrast enhanced lesions mainly in the cerebellum, the occipital and frontal lobes. Which of the below is correct with regard to management of this patient? (Please select 1 option) O Praziquantel 40 mg/kg per day for 6 days and prednisolone 1 mg/kg per day Praziquantel 40 mg/kg per day for 3 days and prednisolone 1 mg/kg per day Praziquantel 40 mg/kg per day for 6 days Praziquantel 60 mg/kg per day for 6 days Praziquantel 60 mg/kg per day for 6 days and prednisolone 1 mg/kg per day encanta Praziquantel 40 mg/kg per day for 6 days Praziquantel 60 mg/kg per day for 6 days Praziquantel 60 mg/kg per day for 6 days and prednisolone 1 mg/kg per day Key Learning Points Infectious Diseases + Praziquantel is the treatment of choice for all Schistosoma species. Explanation Schistosomiasis is a common protozoal infection. Knowledge of howit should be treated is vital. Praziquantel is the treatment of choice for all Schistosoma species. Praziquantel is not licensed for human use in the United Kingdom but is available on named patient basis, Praziquantel 60 mg/kg per day for six days is recommend for S. japonicum with a maximum dose of 5 grams per day with prednisolone 1 mg/kg. Praziquantel 40 mg/kg per day for three days is recommended for S. mansoni and S. haematobium. Since some of the pathology in neuroschistosomiasis is secondary to hypersensitivity reactions there is need to use a steroid, in this case prednisolone 1 mg/kg per day. There is no consensus about when it should be started or stopped. Next Questio encaner ss pel A 47-year-old man with advanced HIV disease presented with fever and obtundation. He has not attended his clinic appointments for several years and is not taking antiretroviral therapy. ACT scan of his brain was normal. A lumbar puncture was performed; microscopy of his cerbrospinal fluid is shown below. What treatment should this patient receive? (Please select 1 option) oO oO Highly active antiretroviral therapy High-dose intravenous aciclovir High-dose intravenous cefotaxime Sulfadiazine + pyrimethamine Amphotericin B + flucytosine encaner ss pel ‘(Highly active antiretroviral therapy High-dose intravenous aciclovir Oo High-dose intravenous cefotaxime O Sulfadiazine + pyrimethamine Amphotericin B + flucytosine Explanation The slide is an India ink stain of CSF that shows typical yeast-like forms of Cryptococcus sp; the thick polysaccharide capsule is highlighted against the darker backgound of black ink. The history is characteristic of cryptococcal disease in AIDS. Treatment is with amphotericin B and flucytosine (SFC); patients then require lifetime suppression with fluconazole. Next Questi | Go to Summary | encaner ss pel A 14-year-old girl presented to hospital with a 24 hour history of nausea, vomiting and abdominal cramps. On examination, she was afebrile but had a tender abdomen with guarding. Three hours after administration of a specific agent, she passed a large quantity of material in her stools (pictured). eh What is the diagnosis? (Please select 1 option cy oO Oo Ascaris lumbricoides Ancylostoma duodenale Necator americanus Strongyloides stercoralis Trichuris trichiura What is the diagnosis? 1 option) Ascaris lumbricoides Key Learning Points Infectious Diseases, Photographic + Ascaris lumbricoides is a large roundworm and is the most common nematode parasite of humans with patients being largely asymptomatic. Explanation Ascaris lumbricoides is a large roundworm, growing up to 35 cmin length, and is the most common nematode parasite of humans. Infected patients are often asymptomatic. Symptoms may develop as a result of pneumonitis caused by the worm's migration through the lungs, obstruction of the gastrointestinal tract or biliary/pancreatic duct obstruction. Piperazine is the treatment of choice in patients presenting with bowel obstruction; mebendazole may be used to treat other infections. Strongyloides stercoralis is a nematode infection. Symptoms may be cutaneous (pruritus due to invasion by the nematodes), pulmonary (eosinophilic pneumonia due to passage through the lungs) or gastrointestinal (abdominal pain). Necator americanus and Ancylostoma duodenale are hookworms (“1 cm in length). The major manifestation of chronic disease is iron- deficiency anaemia. Trichuris trichiura (whipworm) is a helminth approximately 4 cm in length. Infected patients are largely asymptomatic. Further Reading: Division of Parasitic Diseases. Ascariasis. ‘67-year-old woman was admitted to hospital with a tender, hot, right calf. She had a longstanding history of congestive cardiac failure and was known to have chronic oedema of her left leg. The admitting doctor recorded a differential diagnosis of either deep vein thrombosis or cellulitis and started the patient on a treatment dose of low molecular weight heparin and booked a Doppler ultrasound scan of the affected leg. While on the ward overnight, the nurses asked the senior house officer to review the patient as they felt that she was not well. On reviewing her, the SHO found that the patient was confused, febrile 39.0°C, blood pressure 85/60 mmHg, pulse 120 beats per minute and regular, and respiratory rate 32 breaths per minute. The area of erythema that had been present on the right lower leg on admission had extended to the mid-thigh. In recognition of the acutely sick patient, which of the following is correct? (Please select 1 option) © The commonest clinical signs preceding cardiac arrest are tachycardia and hypotension © Arterial blood gas pH is superior to base excess in evaluating metabolic acidosis © The respiratory rate is a highly specific sign predicting underlying physiological disturbance O Ablood lactate level of >1.5 mmol/L is associated with an increased mortality O The degree of base excess disturbance correlates with intravascular fluid requirement Ablood lactate level of 1.5 mmol/L is associated with an increased mortality The degree of base excess disturbance correlates with intravascular fluid requirement Key Learning Points Anaesthetics & ITU, Emergency Medicine, Infectious Diseases + The most common clinical signs prior to a cardiac arrest are respiratory distress (RR >25/min) and altered mental state. Explanation The following can all be used to predict critical illness: * Abnormalities in heart rate * Temperature * Respiratory rate * Urine output, and * Conscious level. All of these have a low specificity but a high sensitivity in detecting underlying physiological disturbance. Most critically ill patients have signs of organ dysfunction that are usually due to underlying tissue hypoxia. The most common clinical signs prior to a cardiac arrest are respiratory distress (RR >25/min) and altered mental state. Over 50% of patients requiring ICU present with with metabolic acidosis many with a raised blood lactate concentration. A raised lactate in associated with a higher mortality. Base excess has been shown to be superior to pH in evaluating metabolic acidosis and in predicting subsequent complications. A base excess of <-4 mmol/L has been associated with a mortality of 50-60%. caeanar ss ba A 60-year-old farmer presents with a six month history of recurrent night sweats and fever. He has lost approximately 5 kg in weight and complains of fatigue. On examination he is afebrile. Heart sounds are normal and his chestis clear. A spleen tip is palpable 4 cm below the left costal margin. The remainder of the examination is unremarkable apart from a faint palpable purpuric rash around both ankles. Investigations show: Haemoglobin WBC Neutrophils Lymphocytes Monocytes Eosinophils Basophils Platelets Serum sodium Serum potassium Serum urea Serum creatinine 121¢/L 75 x10°/L 5.5 x10°/L 2.0 x10°/L 0.05 «109/L 0.01 *10°/L 0.01 «10°/L 110 x10°/L 137 mmol/L. 4.1 mmol/L 9.3 mmol/L 170 pmol/L (130-180) (4-11) (1.5-7) (1,5-4) (0-0.8) (0.04-0.4) (0-0.1) (150-400) (137-144) G.5-4.9) (2.5-7.5) (60-110) Whatis the diagnosis? (Please select 1 option) Serum bilirubin Serum aspartate transaminase Serum alkaline phosphatase Serum albumin Urinalysis Urine microscopy Urine culture O° oO Q fever endocarditis Protein + 12 umol/L (1-22) 98 U/L 4-31) 101 U/L (45-105) 35 e/L (37-49) Red cells seen; no bacteria Negative Non-Hodgkin's lymphoma Brucellosis Renal cell carcinoma Tuberculosis encaner ss pel Serum urea 9.3 mmol/L (2.5-7.5) Serum creatinine 170 pmol/L. (60-110) Serum bilirubin 12 umol/L (1-22) Serum aspartate transaminase 98 U/L (1-31) Serum alkaline phosphatase 101 U/L (45-105) Serum albumin 35e/L (37-49) Urinalysis Protein + Urine microscopy Red cells seen; no bacteria Urine culture Negative Whatis the diagnosis? (Please select 1 option) Q fever endocarditis Non-Hodgkin's lymphoma Brucellosis Renal cell carcinoma Key Learning Points Infectious Diseases + Q fever is a rickettsial zoonotic disease caused by Coxiella burnetii which is usually self limiting. Explanation Q fever is a rickettsial zoonotic disease caused by Coxiella burnetii. Q fever is usually a self-limited respiratory illness due to the inhalation of infected aerosols, especially from animal products. Chronic infection may become established and can manifest as hepati endocarditis. In Q fever endocarditis, the aortic valve is involved in over 80% of cases. Amurmur is not always present, but augmentation of an existing murmur may occur. Low-grade fever (or no fever), signs of heart failure, hepatosplenomegaly, clubbing, arterial emboli, and leukocytoclastic vasculitic rash may also be present. Laboratory tests often show: * Hepatitis * Anaemia * Elevated erythrocyte sedimentation rate * Thrombocytopenia and * Hypergammaglobulinaemia. Microscopic haematuria may be present. The disease may be complicated by immune complex-mediated omerul and arterial emboli. The diagnosis is best made serologically and a phase | antibody titre to Coxiella burnetti (IgG and/or IgA) greater than 1:200 is virtually diagnostic of Q fever endocarditis. Question 48 Question 48 of 193 A 65-year-old female was referred to your practice with a history of having vesicles which later turned into scabs around the inner thigh following the L, dermatome. She is said to have had a burning-like sensation following the same dermatome prior to the development of the vesicles. The vesicles are only on the left side of her body. Whatis the most likely diagnosis? (Please select 1 option) Oo Herpes zoster oO Streptococcal scalded skin syndrome oO Staphylococcus aureus skin infection oO Herpes simplex II O- Tinea inguinalis encaner ss pel Herpes zoster Streptococcal scalded skin syndrome Staphylococcus aureus skin infection Herpes simplex II Tinea inguinalis Key Learning Points Infectious Diseases + Herpes zoster usually has a prodrome pain before the vesicles appear and will then have a dermatomal distribution. Explanation Herpes zoster usually has a prodrome pain before the vesicles appear. It usually follows a particular dermatome but in immune suppression the disease may affect more than one dermatome. Herpes simplex Il is the wrong answer. Herpes simplex II vesicles may appear but they never follow a particular dermatome. Tinea inguinalis is the wrong answer. Tinea inguinalis never forms vesicles but there is the presence of scabs around the groin or any other fold. Streptococcal scalded skin syndrome is the wrong answer. This appears like a burn. It appears as bullae which eventually burst. It could be generalised. Staphylococcus aureus skin infection is the wrong answer. It could sometimes appear as streptococcal scalded skin syndrome, but there are more likely to be pustules. Ccaeaner ss Question 49 of 193 < > Ten individuals are admitted to the Emergency department with profuse vomiting after attending a retirement dinner in a Chinese restaurant. They all ate at roughly 7 pm and became ill at roughly midnight. Nine ate a mixture of meat and rice dishes except one female who ate vegetarian dishes with her rice. Whatis the most likely infective organism? (Please select 1 option) O Clostridium perfringens O Salmonella enteriaitis oO Staphylococcus aureus O- Ecoli O Bacillus cereus Submit Answer encaner ss pel Staphylococcus aureus E. Coli Bacillus cereus Key Learning Points Gastroenterology, Infectious Diseases * Bacillus cereus infection present with profuse vomiting which occurs approximately one to five hours after eating recooked rice. Explanation This is a typical case of Bacillus cereus, with profuse vomiting which occurs approximately one to five hours after eating. In this case it is likely that the rice itself had been infected. Another possibility is Staph. aureus although this is less likely. 1 encaner ss pel Question 50 of 193 < > You are asked to see an 80-year-old lady in an orthopaedic ward. Three weeks previously she had been admitted with a transtrochanteric fracture of her left hip. She was readmitted two days ago with increased pain in the hip and a fever; an infection in the hip prosthesis was confirmed on needle aspiration of the joint. The hemiarthroplasty was promptly surgically excised and the patient was subsequently put on intravenous clindamycin. The orthopaedic firm noticed that this morning the patient has a decreased level of consciousness. They also mention that she has been passing profuse diarthoea since yesterday. There is no medical history of note except for osteoporosis and the patient was fully conscious till yesterday. Examination reveals a frail old lady who looks drowsy and apathetic with a GCS of 12/15. She is clinically dehydrated. Her heart sounds are normal with a weak thready pulse of more than hundred per minute. The ward BP monitor records her BP is 75/50. Her temperature is 35.7°C. Whats the cause of her decreased level of consciousness? (Please select 1 option) Oo Dehydration © Depressive stupor © Septic shock © Pseudomembranous colitis Oca Depressive stupor © Septicshock (EIEE9 6 Pseudomembranous colitis OQ CVA Explanation The patient has many features of septic shock including: * Focus of infection * Diarrhoea * Decreased level of consciousness * Tachycardia, and * Hypothermia. Pseudomembranous colitis is unlikely after just two days of clindamycin. Given the overnight deterioration without focal symptoms, versus a sudden event, a CVAis less likely, and depressive stupor is a diagnosis of exclusion once other causes have been ruled out. Given adequate nursing care, dehydration is not expected to be the cause of tachycardia and hypotension seen here. < A 48-year-old man attends the Emergency department with a two week history of profuse diarrhoea, abdominal pain and flatulence after returning. from a Mediterranean cruise. He describes the stools as being yellow, frothy and offensive smelling. He has lost 5 kg in weight butis afebrile. Stool microscopy is shown below. What is the cause of his diarrhoea? (lease select 1 option) oO Oo Cholera Enteric fever Giardiasis Amoebic dysentry Cryptosporidiosis encaner ss pel Giardiasis Amoebic dysentry Cryptosporidiosis Key Learning Points Gastroenterology, Infectious Diseases, Photographic + Common differentials will include: - Salmonella typhi - Amoebic dystentry Less so - cryptosporidium and Cholera. Norovirus is more likely to present with vomiting. Treatment of Giardia is with metronidazole Explanation The history is typical of giardiasis: abdominal cramps, bloating and flatulence accompanied by yellow, frothy, greasy and offensive- smelling diarrhoea (steatorrhoea). Fever and vomiting are less usual. Diarrhoea may be chronic if undiagnosed and can lead to small bowel malabsorption. The wet mount slide shows cysts of Giardia intestinalis. At higher magnification you would be able to see the characteristic double nuceli in each cyst. Ameobic dysentry causes bloody diarrhoea. Cholera causes profuse, 'rice-water' diarrhoea. Cryptosporidium causes persistent diarrhoea in the young and particularly severe and prolonged diarrhoea in the immunosuppressed, a self limiting infection may occasionally occur in the healthy. Diarrhoea caused by cryptosporidial infection is typically watery. Enteric fever is caused by Salmonella typhi, presentation is classically with fever, malaise, diffuse abdominal pain, and constipation rather than diarrhoea. caeanar ss ba Question 52 of 193 A 34-year-old woman attended the genitourinary medicine clinic complaining of a vaginal discharge. She had seen her GP about it a week ago, who had prescribed her some clotrimazole pessaries following which she had no relief. She denied any itching, but described it as malodorous especially following intercourse. On examination there was a thin white vaginal discharge with a pH of 5.9. Microscopy demonstrated Lactobacilliwith Gram variable rods. Which of the following is the most appropriate treatment? (Please select 1 option) Oo Fluconazole 200 mg daily forfive days Oo Erythromycin 500 mg two times a day for seven days Oo Azithromycin 1 g single dose O Metronidazole 400 mg twice daily for seven days O Ceftriaxone 1 g single dose fweeer] < A 34-year-old woman attended the genitourinary medicine clinic complaining of a vaginal discharge. She had seen her GP about it a week ago, who had prescribed her some clotrimazole pessaries following which she had no relief. She denied any itching, but described it as malodorous especially following intercourse. On examination there was a thin white vaginal discharge with a pH of 5.9. Microscopy demonstrated Lactobacilliwith Gram variable rods. Which of the following is the most appropriate treatment? (Please select 1 option) Fluconazole 200 mg daily forfive days Erythromycin S00 mg two times a day for seven days Azithromycin 1 g single dose Metronidazole 400 mg twice daily for seven days Ceftriaxone 1 g single dose Key Learning Points Infectious Diseases * Bacterial vaginosis may be treated with oral metronidazole 400 mg tds seven days encaner ss pel Explanation This patient describes bacterial vaginosis, a common diagnosis in women of childbearing age. Patients will often describe a malodorous "fishy" thin, milky white discharge which is notitchy. The fishy odour can be elicited by adding 10% potassium hydroxide to the vaginal discharge. The vaginal pH is usually greater than 4.5. Risk factors for bacterial vaginosis include the intrauterine coil device, vaginal douching and number of sexual partners. Bacterial vaginosis has been implicated in the development of pelvic inflammatory diseases. Patients may be asymptomatic. Where identified it should be treated in patients who are symptomatic or pregnant. The first line treatment of bacterial vaginosis in the UK is metronidazole 400 mg twice daily given for seven days. Alternatively, a single dose of oral metronidazole 2 g may be given especially if patient adherence is an issue. Azithromycin is used in the treatment of Chlamydia, and ceftriaxone in the treatment of gonorrhoea. In the US, CDC have updated treatment recommendations for bacterial vaginosis (2010). Metronidazole 500 mg orally twice a day for seven days is the recomended therapy and alternatives include several tinidazole regimens or clindamycin (oral or intravaginal). Additional regimens include metronidazole (750 mg extended release tablets once daily for seven days), or a single dose of clindamycin intravaginal cream, although data on the performance of these alternative regimens are limited. Question 53 of 193 < > A 20-year-old man presents with dysuria and a urethral discharge. Gram staining of the urethral discharge demonstrates neutrophils but no bacteria. Which of the following is the most likely causative organism? (Please select 1 option) Oo Treponema pallidum O Candida albicans O Chlamydia trachomatis O- Chlam dia psittaci O Escherichia coli encaner ss pel Candida albicans Chlamydia trachomatis Chlamydia psittaci Escherichia coli Key Learning Points Infectious Diseases + Different serovars of Chlamydia trachomatis cause either LGV or urethral chlamydia infection. Explanation The most likely diagnosis is infection with Chlamydia trachomatis. Serovars D to K are the most common in the United Kingdom and Present in men with urethri ind in women with urethrit salpingitis and consequent pelivic inflammatory disease. Affected women in particular may be asymptomatic. Chlamydia trachomatisis not visible with Gram stain. (LGV) is caused by Chlamydia trachomatis serovars L1-3 and is endemic in Africa, India, the Caribbean and South East Asia. It presents with a primary genital or skin lesion, followed by lymphadenopathy, flu-like symptoms and occasionally meningitis. Escherichia coliis a common cause of urinary tract infections. Treponema pallidumis the causative organism for syphilis. Chlamydia psittici causes an ic monia in people exposed to respiratory secretions or aerosolised excreta from sick birds. Ccaneaners ss ba < > You are contacted by a nurse from the occupational health tea. She informs you that a junior doctor on the ward has received a needle stick injury from one of your patients. The junior doctor has been fully vaccinated against hepatitis B virus, but she is concerned about the risk of developing HIV from this needle stick injury. The patient in question is a 28-year-old man who has a 10 year history of poorly controlled epilepsy and multiple admissions to ITU as a consequence. He lives with his wife and works as a caretaker. He was admitted due to nausea and vomiting and has been unable to take his medication. Following admission he developed status epilepticus and has been anaesthetised, ventilated and transferred to ITU where he remains unconscious. His HIV status is unknown. The nurse asks if you can perform an HIV test on this patient to guide decisions about post-exposure prophylaxis (PEP). What is the most appropriate course of action? (Please select 1 option) © Take anew blood sample to send for HIV testing © _ Donottest any blood until the patient regains consciousness and discuss with patient at this stage O © Take anHiVtest from patient using point-of-care test using saliva instead of blood O — TestforHiVona sample that has already been taken from this patient O Askthe patient's next-of-kin for permission to take the blood sample What is the most appropriate course of action? (Please select 1 option) oO Take a new blood sample to send for HIV testing © Donottest any blood until the patient regains consciousness and discuss with patient at this stage [ Correct } © TakeanHlVtest from patient using point-of-care test using saliva instead of blood \ Test for HIV ona sample that has already been taken from this patient © Askthe patient's next-of-kin for permission to take the blood sample encaner ss pel Explanation Do not test any blood until the patient regains consciousness and discuss with the patient at this stage. This is correct because this patient has presented the same way multiple times. HIV testing is not necessary to prevent death or serious deterioration. Decisions about PEP must be made without this information at present. The legal position in testing source patients following needlestick injuries is currently not clear. However, British HIV Association guidance (2008) states that: 'If you judge that a patient lacks capacity to consent to an HIV test you should consider whether this is temporary or permanent. If temporary, you should defer testing until the patient regains capacity, unless testing is immediately necessary to save the patient's life or prevent a serious deterioration of their condition.’ GMC guidance at present (26/3/11) states, 'As we understand it, current law does not permit testing the infection status of an incapacitated patient solely for the benefit of a healthcare worker involved in the patient's care.’ In patients with permanent lack of capacity next of kin may be consulted if they have a relevant power of attorney. Taking a new blood sample to send for HIV testing is wrong as it goes against current GMC guidance, as does using point-of-care test using saliva instead of blood. Additionally, point-of-care tests have reduced sensitivity and specificity compared to fourth generation HIV tests. Testing for HIV ona sample that has already been taken from this patient is wrong because this patient has the right to refuse consent to this test. The test should not be done without his being asked for consent. Next Question encaner ss pel A 40-year-old male presents with a cough, weight loss and these lesions on his foot. He has no past medical history, denies drug use, and drinks 25 units of alcohol per week. He has never married, and travels the world in his capacity as the CEO of a successful computer company. What is the likely diagnosis? (Please select 1 option) O Kaposi's sarcoma Oo Squamous cell carcinoma Oo Non-Hodgkin's lymphoma Oo Malignant melanoma © Basal cell carcinoma encaner ss pel What is the likely diagnosis? (Please select 1 option) © Kaposi's sarcoma Gz Squamous cell carcinoma © Non-Hodgkin's lymphoma Malignant melanoma © Basal cell carcinoma Explanation This is the typical lesion of Kaposi's sarcoma. Itis induced by human herpes virus 8 infection in patients with human immunodeficiency virus (HIV). This patient may have contracted HIV on his travels and a full sexual history would be indicated, followed by appropriate counselling and an HIV test. Kaposi's is on the decline due to the advent of highly active antiretroviral therapy. Nemes] encaner ss pel Question 56 of 193 < A17-year-old girl with sickle cell disease presents with pain in her left thigh. Aplain x ray film shows changes consistent with osteomyelitis of the left femur. Which of the following is the most likely causative organism? (Please select 1 option) O O Clostridium perfringens Mycobacterium tuberculosis Listeria monocytogenes Salmonella enteritidis Enterococcus faecium C encaner ss pel 1 Mycobacterium tuberculosis O Listeria monocytogenes © Salmonella enteritidis © Enterococcus faecium Explanation Patients with sickle cell disease have a predisposition to develop osteomyelitis due to Salmonella species. Functional hyposplenism in sickle cell disease also renders sufferers susceptible to infection with encapsulated bacteria (pneumococci, meningococci). Next Quest encaner ss pel A 24-year-old man presented with increasing shortness of breath on exertion. He had been unwell for over two weeks with intermittent fevers and chills. Over the two days preceding his admission to hospital he had developed breathlessness on minimal exertion with a cough productive of blood-stained sputum. He had no previous hospital admissions. He was unemployed and had been homeless until three months previously. He lived in a flat with his girlfriend and their dog. He admitted to regular use of recreational drugs and occasionally injected intravenously. On examination he was febrile 38.5°C. His jugular venous pressure was 9 cm above the sternal angle when lying supine at 45°. His blood pressure was 100/60 mmHg with pulse 104 beats per minute. A pansystolic murmur was audible at the lower left sternal edge. The chest radiograph showed multiple opacities throughout both lung fields. Which of the following antimicrobial regimes is most likely to be effective in treating this patient? (Plea el option) oO IV benzylpenicillin and gentamicin Oo IV amphotericin Oo IV teicoplanin Oo IV benzylpenicillin and metronidazole O w gentamicin and vancomycin IV amphotericin IV teicoplanin IV benzylpenicillin and metronidazole IV gentamicin and vancomycin Ration tire e-1a Key Learning Points Infectious Diseases * Endocarditis from IV use is most commonly associated with Staphylococcus aureus and treated with Gentamicin and Vancomycin. encaner ss pel Explanation This patient has signs and symptoms of right heart endocarditis indicating an urgent need for antibiotics. It is likely that infection has been acquired as a result of intravenous drug use using dirty needles or equipment. The causative organism in most cases of endocarditis from IV use is Staphylococcus aureus. The combination of vancomycin and gentamicin is the only one of the options that would provide adequate antibacterial cover. This combination would also cover MRSA. Some clinicians would use flucloxacillin plus gentamicin as the combination of first choice in such a patient, but since this is not an option this information is of academic interest only. IVDUs are also at risk of developing fungal endocarditis (most frequently non-albicans Candida sp), though this is less common than Staphylococcus aureus. Empiric therapy should start with anti-staphylococcal therapy, as outlined above. If the patient fails to respond or other organisms are isolated from culture specimens, then a change of antibiotic therapy should be made. Teicoplanin has a similar spectrum of antimicrobial activity to vancomycin, but has slightly more anti-streptococcal activity than vancomycin and slightly less anti-staphylococcal activity. It is not a first line agent in the treatment of endocarditis. Benzylpenicillin plus gentamicin is standard treatment of streptococcal endocarditis. The combination of benzylpenicillin plus metronidazole has no role in empiric therapy of infective endocarditis. Ccaeaner sss bl Question 58 of 193 A19-year-old woman came to the hospital with complaints of nausea, vomiting, headache and lethargy. On examination the patient seemed confused and had erythematous macules especially on the lower extremities and the trunk but none on the face. The neck was stiff with a positive Kernig's sign. The CSF comes out with a high pressure during lumbar puncture. Itis turbid with an increased protein level and neutrophilic leukocytosis. The CSF Gram stain revealed Gram negative cocci. What is the causative agent for the above disease? (Please select 1 option) O O S. aureus Neisseria meningitidis Streptococcus pneumoniae H. influenzae Pseudomonas aeruginosa if encaner ss pel 7 S. aureus Neisseria meningitidis Streptococcus pneumoniae H. influenzae Pseudomonas aeruginosa Key Learning Points Infectious Diseases * Neisseria meningitidis are Gram negative cocci. Explanation S. aureus are Gram positive cocci in clusters. Streptococcus pneumoniae are Gram positive cocci in chains. H. influenzae are Gram negative bacilli. Pseudomonas aeruginosa are Gram negative rod-shaped bacteria. encaner ss pel ‘A.44-year-old man presents to the Emergency department with a two week history of progressive dyspnoea, dry cough and fever. Investigations show: Haemoglobin 125 g/L WBC 7.2 10/L Neutrophils 5.9 x10°/L Lymphocytes 0.4 «109/L Monocytes 0.8 x10°/L Platelets 130 x10°/L Sodium 144 mmol/L Potassium 4.1 mmol/L Urea 7.5mmol/L. Creatinine 75 umol/L. Arterial blood gases breathing air show: pH pO PCO? HCO3 742 8.9 kPa/66.8 mmHg 4.7 kPa/35 mmHg 22 mmol/L (430-180) (4-11) (1.5-7) (1.5-4) (0-0.8) (150-400) (137-144) (3.5-4.9) (2.5-7.5) (60-110) (7.36-7.44) (11.3-12.6) (4.7-6.0) (20-28)

You might also like